api.ning.comapi.ning.com/.../LatestNew2.docx  · Web viewNorms develop in one or more of four...

43
Organizational theory and behavior Which of the following is not a topic or concern related to OB? Answer: family behavior What are the three primary behavior determinants of organizational behavior? Answer: individuals, groups, and structure Basing managerial decisions on the best available scientific evidence is called ________. Answer: evidence-based management Organizational behavior is constructed from all of the following disciplines except ________. Answer: physics Organizational behavior is constructed from all of the following disciplines Answer: psychology sociology anthropology social psychology ________ is an effect of increasing globalization. Answer: Jobs moving to nations with low-cost labor ________ is a measure of how organizations are becoming more heterogeneous in terms of gender, race, and ethnicity. Answer: Workforce diversity Approximately ________ % of the U.S. labor force is employed in service industries. Answer: 80 ________ allow(s) people to communicate and work together even though they may be thousands of miles apart. Answer: Networked organizations Situations where an individual is required to define right and wrong conduct are termed ________. Answer: ethical dilemmas ________ is discretionary behavior that is not part of an employee's formal job requirement, but that promotes the effective functioning of the organization. Answer: Organizational citizenship Age, race, gender, ethnicity, religion, and disability status are examples of ________. Answer: surface-level diversity Which of the following is an example of a deep-level diversity difference that might cause challenges between these two co-workers? Answer: Marina is shy and quiet. Amit is outgoing. Exclusion and incivility are difficult forms of discrimination to root out because ________. Answer: they are impossible to observe and may occur unintentionally Research shows that which of the following is likely to decrease as a worker grows older? Answer: likelihood of quitting Which of the following is the most likely explanation for the higher absentee rate of women in the workplace? Answer: Traditionally, women have had the responsibility of caring for home and family. What did the passage of the Americans with Disabilities Act (ADA) require employers to do? Answer: make the workplace accessible to disabled individuals Studies indicate that which of the following tends to decrease with increased tenure? Answer: absenteeism Which of the following do employers need to take into consideration when dealing with transgender employees? Answer: bathroom use Which of the following defines the overall factor of intelligence, as suggested by the positive correlations among specific intellectual ability dimensions? Answer: general mental ability Members of which of the following groups are least likely to turn over in a job? Answer: non-Whites in a predominantly non-White work group http://www.vuzs.net/ Which of the following is an example of an affective attitude? Answer: Anger at being unfairly accused of a wrongdoing.

Transcript of api.ning.comapi.ning.com/.../LatestNew2.docx  · Web viewNorms develop in one or more of four...

Page 1: api.ning.comapi.ning.com/.../LatestNew2.docx  · Web viewNorms develop in one or more of four ways. Which of the following is not one of these ways? a) explicit statements made by

Organizational theory and behaviorWhich of the following is not a topic or concern related to OB?Answer: family behaviorWhat are the three primary behavior determinants of organizational behavior?Answer: individuals, groups, and structureBasing managerial decisions on the best available scientific evidence is called ________.Answer: evidence-based managementOrganizational behavior is constructed from all of the following disciplines except ________.Answer: physicsOrganizational behavior is constructed from all of the following disciplinesAnswer: psychologysociologyanthropologysocial psychology________ is an effect of increasing globalization.Answer: Jobs moving to nations with low-cost labor________ is a measure of how organizations are becoming more heterogeneous in terms of gender, race, and ethnicity.Answer: Workforce diversityApproximately ________ % of the U.S. labor force is employed in service industries.Answer: 80________ allow(s) people to communicate and work together even though they may be thousands of miles apart.Answer: Networked organizationsSituations where an individual is required to define right and wrong conduct are termed ________.Answer: ethical dilemmas________ is discretionary behavior that is not part of an employee's formal job requirement, but that promotes the effective functioning of the organization.Answer: Organizational citizenshipAge, race, gender, ethnicity, religion, and disability status are examples of ________.Answer: surface-level diversityWhich of the following is an example of a deep-level diversity difference that might cause challenges between these two co-workers?Answer: Marina is shy and quiet. Amit is outgoing.Exclusion and incivility are difficult forms of discrimination to root out because ________.Answer: they are impossible to observe and may occur unintentionallyResearch shows that which of the following is likely to decrease as a worker grows older?Answer: likelihood of quittingWhich of the following is the most likely explanation for the higher absentee rate of women in the workplace?Answer: Traditionally, women have had the responsibility of caring for home and family.What did the passage of the Americans with Disabilities Act (ADA) require employers to do?Answer: make the workplace accessible to disabled individualsStudies indicate that which of the following tends to decrease with increased tenure?Answer: absenteeismWhich of the following do employers need to take into consideration when dealing with transgender employees?Answer: bathroom useWhich of the following defines the overall factor of intelligence, as suggested by the positive correlations among specific intellectual ability dimensions?Answer: general mental abilityMembers of which of the following groups are least likely to turn over in a job?Answer: non-Whites in a predominantly non-White work grouphttp://www.vuzs.net/ Which of the following is an example of an affective attitude?Answer: Anger at being unfairly accused of a wrongdoing.Sheila works for a manufacturer of styrofoam trays for school lunches. Last year her son come home from school and told her that the trays she makes go straight to the landfill where they sit for hundreds of years without ever biodegrading. Although this information bothers Sheila, she does not look for another job because she enjoys her very high salary. Which dissonance moderating factor is illustrated by this example?Answer: High dissonance is tolerated because of a high reward.

Page 2: api.ning.comapi.ning.com/.../LatestNew2.docx  · Web viewNorms develop in one or more of four ways. Which of the following is not one of these ways? a) explicit statements made by

In her work in the publishing industry, Vera seeks out new authors who she considers promising. In the past two years she has found a number of new writers whose work she thought was exceptional, and immersed herself in the task of helping them shape their manuscripts for submission to her managers for publishing. Although she was extremely proud of the results, none of the authors she worked with were chosen for publication. Vera believes that the decision not to publish these authors was based on personal rivalries within management, rather than the quality of her writers' work. She is extremely frustrated, dreads coming into work each morning, and is seriously thinking of resigning. How can Vera's job attitudes best be described?Answer: low job satisfaction and high job involvementGuillermo's wife was recently diagnosed with breast cancer. Guillermo had to take a considerable amount of time off work to care for their children, and he was late in completing his portion of a large project. Guillermo never feared that his job was in jeopardy because of his absences or his delay in completion of work. This is an example of which major job attitude?Answer: perceived organizational supportMaria is dissatisfied with the way that her manager treats her. She has quit her job and found a new position with another firm. She has expressed her dissatisfaction through ________.Answer: exitWhich of the following is not an example of OCB?Answer: Discourage a friend from applying for a job at the company.Julia works as a receptionist at a real estate company. Her boss just came in the door and yelled at her, telling her that the front office was a mess and that she needed to get up and clean it immediately. After her boss left the room Julia grabbed three magazines and violently slammed them into the trash can. Which of the following best describes Julia's action?Answer: an emotionPeople think about ________ five times longer than they think about ________ .Answer: negative events, positive eventsWhich of the following statements is not true about moods and exercise?Answer: Everyone benefits the same from an increase in exercise.Which of the following statements is not true about the difference between men and women when it comes to emotional reactions?Answer: Men express emotions more intensely.If an employee expresses organizationally desired emotions during interpersonal transactions, which of the following is being expressed?Answer: emotional laborWhich of the following is an important insight that Affective Events Theory provides?Answer: Emotions provide valuable insights into how workplace hassles and uplifting events influence employee performance and satisfaction.Which of the following is a person's ability to be self-aware, detect emotions in others, and manage emotional cues and information?Answer: emotional intelligenceWhich of the following statements about the determinants of personality is true?Answer: Personality appears to be a result of both hereditary and environmental factors.What does the Myers-Briggs Type Indicator classification of "S or N" stand for?Answer: sensing/intuitiveA book that profiled 13 contemporary business people who created super successful firms including Apple Computer, FedEx, Honda Motors, Microsoft, and Sony found that all 13 were what type of thinkers?Answer: intuitiveA high score in which dimension of the Big Five model predicts good job performance for all occupational groups?Answer: conscientiousnessWhat personality characteristic centers on whether a person is unemotional and pragmatic and believes that ends can justify means?Answer: MachiavellianismDuring an annual review Mitchel made the following assertion: "When I look at myself and my performance I see that what I have achieved is outstanding and has, not surprisingly, won me the admiration and envy of most of my colleagues. I notice that everyone keeps talking about me; they are all just waiting to find out what triumph I will pull off next! In short, I don't just deserve a raise, but need one, since without me, let's face it, the place would simply fall apart." Which of the following is probably the best descriptor of Mitchel's personality?Answer: narcissisticWhich of the following statements about low self-monitors is true?Answer: They tend to pay less attention to the behavior of others than high self-monitors.Which of the following statements about values is true?Answer: Values are fairly stable over time.

Page 3: api.ning.comapi.ning.com/.../LatestNew2.docx  · Web viewNorms develop in one or more of four ways. Which of the following is not one of these ways? a) explicit statements made by

Which of the following terminal values are most common among executives?Answer: self-respect, family security, freedomWhich of the following is not one of Hofstede's five dimensions of national culture?Answer: flexibility versus rigidityTwo people see the same thing at the same time yet interpret it differently. Where do the factors that operate to shape their dissimilar perceptions reside?Answer: the perceiversThe time at which an object or event is seen is an example of what type of factor influencing the perceptual process?Answer: contextWhich of the following is an example of externally caused behavior?Answer: An employee is late because of a flat tire.Your boss never gives you the benefit of the doubt. When you arrived late from lunch, he assumed that you had simply taken too much time. He never considered that the elevators were out and you had to walk up 10 flights of stairs. Your boss is guilty of ________.Answer: fundamental attribution errorWhenever Jane is successful she takes full credit for what has happened, but whenever she is unsuccessful she attributes her failure to bad luck or blames one of her fellow employees. She is guilty of ________.Answer: self-serving biasA committee is made up of 12 managers: three each from the sales, production, accounting, and human resources departments. They read a comprehensive study of the company they work for, and are asked which of its recommendations are most important. In discussion they find that the managers perceive that the most important recommendations are those concerning their own departments. This finding is most likely the result of what type of bias?Answer: selective perceptionYour rating in a job interview is high in contrast to the candidate who was interviewed directly before you, who was rated extremely low. Your own high rating might be partially attributed to which of the following?Answer: contrast effectsAvoiding making decisions on Friday the 13th can be an example of which bias?Answer: randomnessTom knows that the title insurance company made a mistake on his property title. Because of their mistake, his neighbor now has access to an easement road through his back 25 acres. He can't use the acreage for pasture, because his neighbor insists that the gates remain open. The title company has offered a $40,000 settlement. Tom has decided to take the settlement rather than possibly lose in court if he were to sue the title company for more money. Tom's decision is based on ________.Answer: risk aversionGene is regarded by his peers as an extremely creative designer of watercraft. He attributes much of his success to his family: he was raised by a traditional boat builder and from a very early age was surrounded by boats and the people who made them. To what element of the three-component model of creativity does Gene attribute his success?Answer: expertiseAccording to Maslow, when does a need stop motivating?Answer: when it is substantially satisfiedTwo-factor theory suggests that dissatisfaction is caused by extrinsic factors. Which of the following is an example of such a factor?Answer: working conditionsJasmine is trying to gain control of her department. Although she will be greatly compensated if she achieves this aim and will gain control over many subordinates, the main reason she is pursuing this position is that she thinks she can do the job better than her predecessors and wants people to know that this is so. What need would McClelland say primarily drives Jasmine in this case?Answer: the need for achievementTony loves programming. He was on the ground level of the computer revolution. This year his boss has set personal goals for each of the programmers, one of which includes a presentation from each programmer about a project. Tony is very annoyed because he has a tremendous amount of work to do and he'll have to spend the weekend, his personal time, preparing the presentation. Taking into account self-determination theory, why did his boss' goals and the presentation reduce Tony's motivation?Answer: The externally imposed goal feels coercive, causing Tony's intrinsic motivation to suffer.According to the goal-setting theory of motivation, highest performance is reached when goals are set to which level?Answer: difficult but attainable

Page 4: api.ning.comapi.ning.com/.../LatestNew2.docx  · Web viewNorms develop in one or more of four ways. Which of the following is not one of these ways? a) explicit statements made by

Jessica believes that she received an appropriate salary increase this year but she does not believe that the company's methods for determining salary increases were fair. She believes that there is a problem with the firm's ________ justice.Answer: proceduralNikhil is a design manager for a production company. He independently assigns teams, chooses projects, researches trends, and is able to work from home two to three days a week. He provides a status report to the design director once a month. Based on this description, it's safe to say that Nikhil's job is high in which job characteristic?Answer: autonomyWhich of the following is the principal drawback from job sharing?Answer: difficulty in finding compatible partnersResearch studies conducted on the participation-performance relationship show that the use of participation ________.Answer: has only a modest influence on employee productivityWhich of the following is not one of the most widely used variable-pay programs?Answer: flat payWhat is the term used for the situation where employees are allowed to pick and choose from among a menu of benefit options?Answer: flexible benefitsWhich of the following is a form of recognition?Answer: personal congratulationsDiversity training almost always includes all of the following excepta) interaction with representatives from diverse groupsb) engaging in various activities to change attitudesc) emphasis on preservation of traditional valuesd) self-evaluation or self-assessmente) confronting own ethnic and cultural stereotypesAnswer c) emphasis on preservation of traditional valuesThe measure of the extent to which a society accepts the fact that power in institutions and organizations is unequally share is calleda) power distanceb) bureaucracyc) organizational power theoryd) collectivisme) institutional distribution theoryAnswer a) power distanceTed has expressed that among his life goals are to write a book, volunteer extensively, and travel abroad. These are examples of Ted'sa) fundamental valuesb) core valuesc) global valuesd) terminal valuese) instrumental valuesAnswer d) terminal valuesIndividualism is the degree to which people prefer to act as a ________, whereas collectivism is the equivalent of ________.a) power brokers; low self-esteemb) individuals; group behaviourc) individuals; low individualismd) power brokers; low individualisme) individuals; uncertainty avoidanceAnswer c) individuals; low individualismWhich of the following is not considered to be a way to deal with excessive stress?a) tennis matchb) swimmingc) riding a biked) jogginge) aerobicsAnswer a) tennis matchWhich of the following factors is not part of job satisfaction?a) fair and equitable rewardsb) organizational commitment

Page 5: api.ning.comapi.ning.com/.../LatestNew2.docx  · Web viewNorms develop in one or more of four ways. Which of the following is not one of these ways? a) explicit statements made by

c) opportunity to use skills and abilitiesd) immediate supervisor who is supportive and understandinge) supportive working environmentAnswer b) organizational commitmentWhy have non-aboriginals traditionally view aboriginal values as "an impediment to economic development" and organizational effectiveness?a) low rating on uncertainty avoidanceb) incompatible gender differentiationc) incompatible power distanced) low rating on assertivenesse) time orientation and consensus decision makingAnswer e) time orientation and consensus decision makingAs a union member, Kim is most likely to favour which of the following terminal values the most?a) family securityb) freedomc) couraged) a world of peacee) honestyAnswer a) family securityWhich of the following topics is generally considered to be outside the field of OBa) employment turnoverb) job satisfactionc) therapyd) absenteeisme) productivityAnswer c) therapyVarious challenges exist at the organizational level. Which of the following would not be considered an organizational challenge?a) appreciating the diverse workforceb) overall productivity and outputc) hiring and developing effective employeesd) developing and implementing successful managerial intervention techniquese) competing in global marketsAnswer a) appreciating the diverse workforceMotivation is an important_________ variablea) departmental-levelb) individual-levelc) industry-leveld) organization systems-levele) group-levelAnswer b) individual-levelWhy do organizations that have OCB outperform those that do not?a) increased job satisfactionb) increased opportunity for promotionsc) employees are easier to supervised) promotion of effective functioning of the organizatione) frequent recruitment, selection and trainingAnswer d) promotion of effective functioning of the organizationAll of the following statements are true of low self-monitors except:a) they can't disguise themselves wellb) they tend to pay less attention to the behaviour of others than high self-monitorsc) they tend to display their true dispositions and attitudesd) they tend to pay closer attention to the behaviour of others than high self-monitorse) they have high behavioural consistency between who they are and what they doAnswer d) they tend to pay closer attention to the behaviour of others than high self-monitors_______ is the tendency to attribute one's own characteristics to other peoplea) selectionb) projection

Page 6: api.ning.comapi.ning.com/.../LatestNew2.docx  · Web viewNorms develop in one or more of four ways. Which of the following is not one of these ways? a) explicit statements made by

c) interpretationd) stereotypinge) halo effectAnswer b) projectionGeneration X is characterized by which of the following common values?a) belief in, and practise of, the Christian moral codeb) strong concern for the environmentc) intimidated by changed) adaptability and concern with personal imagee) belief in order and authorityAnswer d) adaptability and concern with personal imageAccording to Michael Adams, "the elders", one of the broad age groups of adult Canadians, are characterized by all of the following except:a) living by the golden ruleb) authorityc) belief in orderd) disciplee) autonomyAnswer e) autonomyIn contrast to the hierarchy of needs, the ERG theory demonstrates:a) that lower-level needs must be satisfied before higher-level needsb) that interpersonal relationships are the most pressing need to be satisfiedc) that ERG is less consistent with our knowledge of individual differences than other needs theoriesd) that more than one need may be working at the same timee) that no more than one need may be working at one timeAnswer d) that more than one need may be working at the same timeThe drive to become what one is capable of becoming constitutes which level of Maslow's hierarchy of needs?a) socialb) physiologicalc) safetyd) self-actualizatione) esteemAnswer d) self-actualization___________ justice is the perceived fairness of process used to determine the distribution of rewards.a) Proceduralb) Distributivec) Relatived) Interactionale) OrganizationalAnswer a) ProceduralAccording to expectancy theory, for employees to be motivated:a) rewards are irrelevant and performance is the key driver for motivationb) performance must be indirectly linked to rewardsc) their compensation must be incrementald) performance must be directly linked to rewardse) they can expect to work less but still find financial benefitsAnswer e) they can expect to work less but still find financial benefitsWhich of the following is outside the scope of role expectations?a) It is one's own view of how one should behave or actb) role stereotypes are derived from expectationsc) it is how others believe one should act in a given situationd) behaviour can be determined from expectationse) the psychological contract aids in identifying mutual role expectations by employee and employerAnswer a) It is one's own view of how one should behave or actWhich of the following teams primarily use computer technology to tie together physically dispersed member in order to achieve a common goal?a) virtual

Page 7: api.ning.comapi.ning.com/.../LatestNew2.docx  · Web viewNorms develop in one or more of four ways. Which of the following is not one of these ways? a) explicit statements made by

b) quality circlec) cross-functionald) problem-solvinge) skunkworksAnswer a) virtualRetranslating a sender's communication message describes:a) decodingb) the channelc) feedbackd) the messagee) encodingAnswer a) decodingYour supervisor embarrasses you by constantly telling sexual jokes and making sexual comments. Although he has never directly asked you to sleep with him, you feel threatened and are so uncomfortable that it is difficult for you to do your job.A basic problem in this situation is that:a) your supervisor probably has nothing better to dob) your perception is probably very different from your supervisor'sc) there is an unequal power relationship between your supervisor and youd) this is not clearly sexual harassment, since he has not requested that you give him sexual favourse) your clothes are probably too suggestiveAnswer c) there is an unequal power relationship between your supervisor and youWhich behavioural science discipline contributes to OB's understanding of group decision-making processes?a) social psychologyb) political sciencec) psychologyd) anthropologye) sociologyAnswer a) social psychologyHow does selective perception works as a shortcut in judging other people?a) such observations allow us to arrive at warranted judgements quicklyb) It allows us to "speed-read" without the risk of inaccurate conclusionsc) Bits and pieces of what we observe allow us to judge people with complete accuracyd) bits and pieces of what we observe are selectively chosen according to our interests and backgrounde) Bits and pieces of what we observe are chosen randomlyAnswer d) bits and pieces of what we observe are selectively chosen according to our interests and backgroundJack was disappointed that many companies and organizations do not offer training programs or other incentives to accommodate diversity. He found all of the following were reasons that cross-cultural training was not considered except:a) top management doubted the effectiveness of cross-cultural trainingb) senior managers believed that managerial skills are transferable across cultures - the management process is universalc) some companies utilized experts to advise management on diversity issuesd) performance and productivity can be significantly affectede) some companies simply ignore cultural differencesAnswer d) performance and productivity can be significantly affectedWhich of the following is an example of an MBO objective?a) telephone orders should be processed promptlyb) quality of all products should increase substantiallyc) company costs should be decreased as much as possibled) new customers should be brought in on a regular basise) sales should be increased by two percentAnswer e) sales should be increased by two percentA telephone installer who schedules work for the day, makes visits without supervision, and decides the most effective techniques for installation is an example of:a) high significanceb) high identityc) high varietyd) high feedback

Page 8: api.ning.comapi.ning.com/.../LatestNew2.docx  · Web viewNorms develop in one or more of four ways. Which of the following is not one of these ways? a) explicit statements made by

e) high autonomyAnswer e) high autonomyAfter which stage of a group's development is there a relatively clear hierarchy of leadership within a group?a) normingb) goal settingc) performingd) storminge) formingAnswer d) stormingThe more levels a message must go through to reach the bottom of the organizational hierarchy, the more likely that:a) there will be a greater need for team developmentb) the power of managers will be dilutedc) filtering will occurd) subordinates will be discontentede) written memos will be usedAnswer c) filtering will occurMost studies confirm that the concept of ______ is central to understanding sexual harassmenta) locus of controlb) powerc) common sensed) sexe) politicsAnswer b) powerMembers of groups must contend with various challenges on the job. Which of the following would not be considered a group challenge:a) developing an atmosphere of team spirit and supportb) understanding workforce diversityc) working with others from different culturesd) working with others who do not share a common ethnic backgrounde) empowermentAnswer e) empowermentJane Simpson rates low on conscientiousness. You should expect would:a) be comfortable with solitudeb) be easily distractedc) be achievement orientedd) find comfort in the familiare) be nervous, depressed, and insecureAnswer b) be easily distractedWhich activity has long been recommended by physicians as a way to deal with excessive stress levels?a) boxingb) walkingc) rugbyd) footballe) soccerAnswer b) walkingOne reason to consider graphic rating scales is:a) quality of resultb) their superiority to all other forms of evaluationc) they require little time to completed) accuracye) breadth of informationAnswer d) accuracyNorms develop in one or more of four ways. Which of the following is not one of these ways?a) explicit statements made by a group memberb) critical events in the group's historyc) setting behaviour for every conceivable situationd) primacy - the first behaviour pattern that emerges

Page 9: api.ning.comapi.ning.com/.../LatestNew2.docx  · Web viewNorms develop in one or more of four ways. Which of the following is not one of these ways? a) explicit statements made by

e) carry-over behaviours from past situationsAnswer c) setting behaviour for every conceivable situationAll communication occurs with a __________, and violation of the _____ may create additional problems in sending and receiving messages.a) context; contextb) filter; grapevinec) message; channeld) channel; channele) context; feedback loopAnswer a) context; contextApplication of organizational behaviour concepts is most useful within organizations or employment situations onlyAnswer FInterviewers make perceptual judgements during interviews that affect whether an individual is hired.Answer TDissatisfied employees are more likely to miss work, but the correlation is only moderateAnswer TThe choice of the performance evaluation system as little effect on employee behaviourAnswer FFlextime tends to reduce absenteeismAnswer TRole expectation would assist in mentorship programsAnswer TInformation overload is a conditio in which information inflow exceeds an individual's processing capacityAnswer TInformation power is derived from access to and control over informationAnswer TLearning, perception, and personality have been OB topics whose contributions have generally come from psychologyAnswer TIf you expect older workers to be unable to learn a new job skill, that is probably what you will perceive, whether it is accurate or notAnswer T

 

All of the following contribute to a toxic organization except:a) terrorismb) downsizingc) sudden death of a key managerd) organizational changee) strong competition from a start-up companyAnswer a) terrorismAffective conflict is:a) impersonal and based on issues rather than a personalityb) emotional and aimed at a person rather than an issuec) a divergent hierarchy of goalsd) a consequence of the requirements of the jobe) related to differences in perspectivesAnswer b) emotional and aimed at a person rather than an issueRecent research studies in gender differences in negotiating styles and effectiveness show thana) there are many reliable conclusionsb) there are few reliable conclusionsc) women are better negotiatorsd) no differences existe) women are more confidentAnswer b) there are few reliable conclusions

Page 10: api.ning.comapi.ning.com/.../LatestNew2.docx  · Web viewNorms develop in one or more of four ways. Which of the following is not one of these ways? a) explicit statements made by

Which of the following is also a potential conflict?a) where group or department members are olderb) intergroup fighting for control of resourcesc) lower turnover that does not affect group performanced) structural conflict is not a possible issuee) specific direction/definition of responsibility for actionsAnswer b) intergroup fighting for control of resourcesWhich of the following conflict-handling methods is best described as win-win?a) compromisingb) problem solvingc) competingd) avoidinge) accommodatingAnswer b) problem solvingJanet & Phillip have decided to bring in a neutral third party to facilitate a negotiated solution This person is referred to as a(n):a) liaisonb) mediatorc) conciliatord) consultante) arbitratorAnswer b) mediatorWhich of the following is not consistent with the definition of a ritual?a) sequence of activitiesb) storiesc) repetitiond) key valuese) which people are importantAnswer b) storiesWhich of the following socialization processes is best matched with apprenticeships?a) randomb) informalc) divestitured) seriale) collectiveAnswer d) serialWhich of the following is inconsistent with feeling cut out for a networked culture?a) are highly task performance-orientedb) possess good social skillsc) thrive in a realized and convivial atmosphered) prefer close working relationshipse) empatheticAnswer a) are highly task performance-oriented__________ is the degree to which employees are expected to work with precision, analysis, and attention to detail.a) Institutionalizationb) Outcome orientationc) Attention to detaild) People orientatione) SocializationAnswer c) Attention to detailAccording to path-goal theory, an approach focused on providing specific guidance and establishing work schedules and rules is referred to as:a) charismaticb) participativec) supportived) institutionale) directiveAnswer e) directive

Page 11: api.ning.comapi.ning.com/.../LatestNew2.docx  · Web viewNorms develop in one or more of four ways. Which of the following is not one of these ways? a) explicit statements made by

Research findings indicate that when an employee has a clear understanding of how to perform a task and reach a goal a __________ leader will increase employee satisfactiona) task-orientedb) people-orientedc) technical-orientedd) production-orientede) goal-orientedAnswer b) people-orientedIf trait research had been successful, we would _____, whereas if behavioural studies were correct, we would ______a) change jobs to suit people, change people to suit jobsb) teach people certain traits; teach people certain behavioursc) hire only women; hire either men or womend) teach people managerial principles; teach people certain behaviourse) select the right person for the job; train leadersAnswer e) select the right person for the job; train leadersUtilitarianism is when decisions are made to provide thea) greatest good for the least privilegedb) least good for the least numberc) greatest good for the greatest numberd) greatest good or the decision makerse) greatest good for the best performersAnswer c) greatest good for the greatest numberThe tendency to overstate the risk of one activity while simultaneously underestimating the risk of an alternative is to employa) regression to the meanb) escalating commitmentc) the representative heuristicd) framing modele) the availability heuristicAnswer e) the availability heuristicBrainstorming is:a) a technique that tends to restrict independent thinkingb) used when no other method is availablec) a process for generating ideasd) used mainly when group members cannot agree on a solutione) used to build group cohesivenessAnswer c) a process for generating ideasWhich of the following concepts includes only formal authority, that is, the rights inherent in one' positiona) complexityb) formalizationc) departmentalizationd) centralizatione) technologyAnswer d) centralizationIf a job is highly formalized, it would not include:a) high employee job discretionb) clearly defined procedures on work processesc) consistent and uniform outputd) a large number of organizational rulese) an explicit job descriptionAnswer a) high employee job discretionYou have eliminated horizontal, vertical, and external barriers. You are operating as aa) matrixb) decentralized organizationc) team structured) boundary-less organizatione) virtual organizationAnswer d) boundary-less organization

Page 12: api.ning.comapi.ning.com/.../LatestNew2.docx  · Web viewNorms develop in one or more of four ways. Which of the following is not one of these ways? a) explicit statements made by

Which of the following is generally considered outside the methods associated with changing organizational structures?a) creation of a matrix designb) use of flexible work hoursc) modification of compensation systemsd) use of automatione) redefining job descriptionsAnswer c) modification of compensation systemsThe re-engineering process is charactered by by a series of beliefs and actions. Which of the following is not part of the re-engineering process?a) realizing significantly large increases in performanceb) employees discretion about how process will be handled is driven by top managementc) process is autocratic and non-democraticd) discarding irrelevant practises and starting over againAnswer b) employees discretion about how process will be handled is driven by top managementThe built in mechanisms that an organization has to produce stability are referred to as:a) habitb) threat to expertisec) selective information processingd) structural inertiae) economic factorsAnswer d) structural inertiaWhich of the following facts is correct when it comes to workplace commitment?a) the younger the employee, the higher the commitment levelb) canadians are more committed to to their job by almost double when compared to Americansc) a loosely structured organization increases commitmentd) almost 75% of Canadians are satisfied with their supervisorse) working canadians are less committed to their employer than they were in 1991Answer e) working canadians are less committed to their employer than they were in 1991In third-party negotiations, a third party with the authority to dictate an agreement is known as a(n):a) mediatorb) conciliatorc) arbitratord) counselore) consultantsAnswer c) arbitratorWhen it comes to matching people with cultures, research has shown that the following two dimensions underlie organizational culturea) sociability, solidarityb) sociability, adaptabilityc) metamorphosis, adaptabilityd) educational training, adaptabilitye) cooperativeness, flexibilityAnswer a) sociability, solidarityWhich type of group would have the greatest potential for interpersonal conflict?a) brainstormingb) heterogeneousc) electronicd) interactinge) nominalAnswer c) electronicIf you have a narrow span of control, you have what type of organization?a) wideb) multi-levelc) shortd) matrixe) tallAnswer e) tall

Page 13: api.ning.comapi.ning.com/.../LatestNew2.docx  · Web viewNorms develop in one or more of four ways. Which of the following is not one of these ways? a) explicit statements made by

Which of the following correctly identifies why individuals may resist change?a) habit, security, economic factors, fear of the known, selective information processingb) habit, security, econcomic factors, fear of the unknown, selective information processingc) habit, security, cultural factors, fear of the unknown, selective information processingd) habit, security, economic factors, fear of the unknown, reckless information processinge) habitat, security, economic factors, fear of the unknown, selective information processingAnswer b) habit, security, econcomic factors, fear of the unknown, selective information processingWhich of the following falls outside the general definition of conflict?a) difference over interpretations of factsb) incompatibility of goalsc) miscommunication of informationd) disagreement on behavioural expectationse) commonly accepted organizational practicesAnswer e) commonly accepted organizational practicesWhich of the following leadership theories assumes that leaders have specific characteristics?a) behaviouralb) path-goalc) contingencyd) traite) situationalAnswer e) situationalIf you choose to attempt to explain ethical and unethical behaviour, you will include:a) project management principlesb) escalation of commitmentc) management principlesd) paradigm shifte) stages of moral developmentAnswer e) stages of moral development"To what degrees are tasks subdivided into separate jobs?" the answer to this question is provided by:a) span of controlb) chain of commandc) work specializationd) work diversitye) formalizationAnswer c) work specializationHow are outside consultants disadvantaged as change agents?a) prone to initiating too few reformsb) inadequate understanding of the organizations culture and historyc) inadequate understanding of the basic organizational behaviourd) prone to inaction through inadequate understandinge) less objective than staffAnswer b) inadequate understanding of the organizations culture and historyAlthough men and women are targets of disrespect and rudeness in equal numbers,a) men instigate the rudeness 30% of the timeb) women instigate the rudeness 70% of the timec) men instigate the rudeness 70% of the timed) women are less likely to admit that have made mistakese) women are more likely to instigate conflictAnswer c) men instigate the rudeness 70% of the timeWhen upper management struggles to maintain the status quo, regardless of the outcome, this is an example of toxic managerial behaviourAnswer FWhen an organization is new, acceptable modes of behaviour are now always readily apparent to its membersAnswer TIf behavioural theories of leadership are valid, individuals can be trained to be leaders.Answer T

Page 14: api.ning.comapi.ning.com/.../LatestNew2.docx  · Web viewNorms develop in one or more of four ways. Which of the following is not one of these ways? a) explicit statements made by

Rationality assumes that the criteria and alternatives can be ranked according to their importanceAnswer TThe matrix structure is most widely practiced in small businesses in which the manager and the manager are one and the sameAnswer FRe-engineering requires management to reorganize around vertical processesAnswer FTeams experiencing affective conflict have poorer decisions and lower levels of acceptance of the decisionAnswer TStrong cultures are made up of values that are intensely held and widely sharedAnswer TA transformational leader will attempt to instil in followers the ability to question established views as well as those being created by the leaderAnswer TGroup decisions are almost always more inefficient than individual decisionsAnswer TMGT502 Organizational Behaviour Solved MCQs  Set 3

Don has a reputation for not being "straight" with his employees. Their view is that he has a tendency to tell them what they want to hear rather than what is actually happening. This tendency is known asa) filteringb) defensivenessc) information overloadd) selective perceptione) channellingAnswer a) filtering

An example of a(n) _______ communication network is when a checkout clerk reports to a department manager, who in turn reports to a store manager, who reports to a regional manager.a) wheelb) lateralc) chaind) all-channele) "Y"Answer c) chain

Suggestion boxes, employee attitude surveys, and grievance procedures are examples ofa) group process communicationb) upward communicationc) horizontal communicationd) managerial communicatione) organizational structureAnswer b) upward communication

It it easy to tell when Donald isn't listening by the way he looks away when you are speaking to him and he begins to tap his pen. Donald is communication througha) lateral communicationb) proxemicsc) social distanced) nonverbal communicatione) selective perceptionAnswer d) nonverbal communication

The communication that is used by group leaders and managers to assign goals, point out problems that need attention, and provide job instructions isa) group communicationb) lateral communicationc) downward communication

Page 15: api.ning.comapi.ning.com/.../LatestNew2.docx  · Web viewNorms develop in one or more of four ways. Which of the following is not one of these ways? a) explicit statements made by

d) directional communicatione) informal communicationAnswer c) downward communication

In the electronic age, all employees can theoretically bea) home-based workersb) on call 24/7c) unemployedd) outsourcede) highly trained expertsAnswer b) on call 24/7

Which of the following is not generally considered to be an aspect of listening effectivelya) avoiding eye contactb) avoiding interruptingc) asking questionsd) over talkinge) avoiding distractionsAnswer a) avoiding eye contact

Face-to-face communication scores highest in terms of channel richnessAnswer T

High-performing managers tend to communicate non routine messages efficiently through channels that are lower in richnessAnswer F

When we think of managers communicating with employees, the downward flow pattern is the one we usually have in mindAnswer T

Astute communicators watch for gaps, pauses, and hesitationsAnswer T

Paraphrasing is an effective feedback technique in the communication processAnswer T

A facial expression can indicate something different from the simultaneously sent verbal messageAnswer T

Rumours flourish in ambiguous situationsAnswer T

Cultural norms have nothing to do with the physical distance an individual requiresAnswer F

Which of the following is likely to generate the lowest number of ideas?a) nominal group techniqueb) electronic meetingsc) computer-assisted groupd) face-to-face- interacting groupse) brainstormingAnswer d) face-to-face interacting groups

The most recent approach to group decision making blends the nominal group technique with computer technology. It is known asa) computerized decision makingb) the electronic meetingc) electronic maild) electronic participation

Page 16: api.ning.comapi.ning.com/.../LatestNew2.docx  · Web viewNorms develop in one or more of four ways. Which of the following is not one of these ways? a) explicit statements made by

e) electronic database handlingAnswer b) the electronic meeting

What Don doesn't like about working in groups is that they take a lot of time to do things, much more time than it would take him to do things on his own. Thus, in terms of usage of time, his view is that groups area) as efficient as individualsb) more effective than individualsc) less efficient than individualsd) more efficient than individualse) less effective than individualsAnswer c) less efficient than individuals

Individuals who have progressed to higher levels of moral development will place increased value on the rights of others. These individuals area) predisposed to use their influence to disregard the organization's value systemb) likely to value the majority opinionc) likely to challenge organizational practices that they feel or believe are wrongd) likely to follow an organization's rules and procedurese) strongly influenced by their peers to conformAnswer c) likely to challenge organizational practices that they feel or believe are wrong

Norma always tries to make decisions that will result in the greatest good for the greatest number of people. This approach is known asa) ethical decision makingb) moralityc) groupthinkd) whistle blowinge) utilitarianismAnswer e) utilitarianism

Which of the following conditions would probably not lead to intuitive decision making?a) when facts don't clearly point the way to gob) when a high level of certainty existsc) when there is pressure to come up with the right decisiond) when facts are limitede) when time is limitedAnswer b) when a high level of certainty exists

Todd has a tendencty to rely too much on the initial information at his disposal about any given issue such that he fails to factor in adequately new information. This is known asa) anchoring biasb) overconficence biasc) performance biasd) confirmation biase) availability biasAnswer a) anchoring bias

Looking for a solution that is satisfatory and sufficient is calleda) satisficingb) optimizingc) seeking an implicit favourited) sub-optimizinge) simplifyingAnswer a) satisficing

Which type of decision-making group is most committed to the group solution?a) interacting

Page 17: api.ning.comapi.ning.com/.../LatestNew2.docx  · Web viewNorms develop in one or more of four ways. Which of the following is not one of these ways? a) explicit statements made by

b) computer-assisstedc) electronicd) brainstorminge) nominalAnswer a) interacting

A disparity between some desired state and actual condition is define as a(n)a) criterionb) judgementc) problemd) attributione) decisionAnswer c) problem

Ben has noticed that after his team discusses any issue, the conservative people become even more cautious and the aggressive people get even bolder. The phenomenon is know asa) groupshiftb) groupthinkc) consistency biasd) perpetual biase) randomness errorAnswer a) groupthink

Carol has designated Kevin to be the group's "Devil's Advocate". His job was to challenge the majority opinion on each issue and to offer divergent perspectives. Which group decision making problem is Carol attempting to avoid?a) consistency biasb) randomness errorc) groupshiftd) perceptual biase) groupthinkAnswer e) groupthink

Is has been well documented that individuals escalate commitment to a failing course of action when they view themselves as responsible for the failureAnswer T

Groups usually make more risky decisions than do individualsAnswer T

The rational decision-making model assumes rationality and that the alternatives that yields the highest perceived value will be chosenAnswer T

The bounded rationality model assumes that the decision maker will simplify the problemAnswer T

Group decision are more likely to be accepted than individual decisionsAnswer T

Intuitive decision making is a conscious process created out of experienceAnswer F

What is considered unethical in one country may be considered ethical in another cultureAnswer T

Illusion of unanimity and rationalized resistance are symptoms of the groupthink phenomenonAnswer T

Page 18: api.ning.comapi.ning.com/.../LatestNew2.docx  · Web viewNorms develop in one or more of four ways. Which of the following is not one of these ways? a) explicit statements made by

The two leaders in your department are very different. John believes that he should guide and motive by clarification of the roles and tasks of his subordinates. Alan feels that his role should be to provide individualized consideration and intellectual stimulation. Most of the workers believe that both leaders are well trained, but Alan has a special "spark" that employees feel when they work with him.John is which type of leader?a) laissez-faireb) teamc) charismaticd) tranformationale) transactionalAnswer e) transactional

Blake & Mouton were responsible for the development of thea) Leader-member exchange theoryb) BCG matrixc) Delphi techniqued) path-goal modele) managerial gridAnswer e) managerial grid

Your supervisor has been studying leadership and has described so many theories to you that you are really confused. You ask her to explain the similarities of the many theories so that you can understand the relationship between them.What other term might be used to describe the concept of "task"?a) employee-orientedb) relationship-orientedc) initiating structured) consideratione) supportiveAnswer c) initiating structure

Although organizations require a combination of strong leadership and strong management, the process of leadership does have specific characteristics. Which of the following best describes the role of leadership?a) leaders are micromanagersb) leaders have no vision- managers do thatc) leaders let managers deal with changed) leaders let individuals deal with their own personal hurdles by leaving them on their owne) leaders innovate for the entire organizationAnswer e) leaders innovate for the entire organization

Doug's belief that employees should interact as little as possible to achieve efficiency can be characterized, according to the managerial grid, asa) authority-obedianceb) organization managementc) country club managementd) team managemente) impoverished managementAnswer a) authority-obediance

The two leaders in your department are very different. John believes that he should guide and motive by clarification of the roles and tasks of his subordinates. Alan feels that his role should be to provide individualized consideration and intellectual stimulation. Most of the workers believe that both leaders are well trained, but Alan has a special "spark" that employees feel when they work with him.That special "spark" that people notice when they work with Alan is termeda) laissez-fairb) transcendent

Page 19: api.ning.comapi.ning.com/.../LatestNew2.docx  · Web viewNorms develop in one or more of four ways. Which of the following is not one of these ways? a) explicit statements made by

c) charismad) structuree) transactionalAnswer c) charisma

Your supervisor has been studying leadership and has described so many theories to you that you are really confused. You ask her to explain the similarities of the many theories so that you can understand the relationship between them.What other terms might be used to describe the concept of "people"a) consideration, employee-oriented, and supportiveb) goal-setting, planning, and supportivec) supportive, directive leadership, and relationship-orientedd) consideration, relationship-oriented, and initiating structuree) directive leadership, production-oriented, and supportiveAnswer a) consideration, employee-oriented, and supportive

Lily does a great job of inspiring her employees to go beyond their own self-interests for the good of the organization. She thus is known as a _______ leadera) transactionalb) transformationalc) competentd) supportivee) participativeAnswer b) transformational

Cora has highly motivated staff that enjoys challenges. According to Path-Goal Theory, the best leadership style for her would bea) directiveb) achievement-orientedc) supportived) charismatice) participativeAnswer b) achievement-oriented

Which of the following did the Ohio State studies identify as the dimensions by which managers can be characterized?a) the dimensions found were consideration and initiating structureb) leaders cannot effectively perform both dimensions of leadershipc) laissez-faire leadership was most effectived) middle-of-the-road management produces satisfactory resultse) the dimensions found were achievement orientation and participationAnswer a) the dimensions found were consideration and initiating structure

The type of leaders who guide or motivate their followers in the direction of established goals by clarifying role and task requirements area) employee-oriented leadersb) charismatic leadersc) transactional leadersd) process-oriented leaderse) transformational leadersAnswer c) transactional leaders

According to the Ohio State studies, the extent to which a leader shares mutual trust and respect for his or her employees is referred to asa) task-oriented behaviourb) concern for productionc) initiating structured) consideratione) employee-oriented behaviorAnswer d) consideration

Page 20: api.ning.comapi.ning.com/.../LatestNew2.docx  · Web viewNorms develop in one or more of four ways. Which of the following is not one of these ways? a) explicit statements made by

What theory proposes effective leaders adapt their leadership style according to how willing and able a follower is to perform task?a) certainty modelb) path-goal theoryc) contingency modeld) task-structure modele) situational leadership modelAnswer e) situational leadership model

The University of Michigan studies define which leader as one who takes personal interest in the needs of his or her subordinates?a) humanisticb) personalc) employee-orientedd) achievement-orientede) country clubAnswer c) employee-oriented

According to path-goal theory, a leader who sets challenging goals and seeks improvements in performance is referred to asa) charismaticb) institutionalc) participatived) achievement-orientede) directiveAnswer d) achievement oriented

The leadership model that integrates the expectancy model of motivation with the Ohio State leadership research isa) Fiedler'sb) leader-member exchangec) autocratic-democraticd) path-goale) leader-pericipationAnswer d) path-goal

Leaders who believe that they are empowering their employees by giving up their responsibility for leading are calleda) laissez-faire leadersb) transformational leadersc) situational leadersd) transcendent leaderse) transactional leadersAnswer a) laissez-faire leaders

Which f the following statements is inconsistent with successful leadership behaviour patterns?a) effective leaders are more concerned with the people they lead then about getting the job doneb) Two dimensions of leader behaviour have been labelled as initiating structure and considerationc) an individual scoring high on one dimension of leader behaviour will not necessarily score low on the other other dimensiond) the two major dimensions of leader behaviour focus on people and taskse) pioneering work in this area was done at the University of Michigan and the Ohio State UniversityAnswer a) effective leaders are more concerned with the people they lead then about getting the job done

The University of Michigan studies identify leaders who emphasize interpersonal relations asa) initiating-orientedb) contingency-orientedc) production-orientedd) employee-orientede) relationship-orientedAnswer d) employee-oriented

Page 21: api.ning.comapi.ning.com/.../LatestNew2.docx  · Web viewNorms develop in one or more of four ways. Which of the following is not one of these ways? a) explicit statements made by

Fred is facing a decision where it is critical that his staff buy into it. According to Path-goal Theory, the best leadership style for him would bea) directiveb) supportivec) achievement-orientedd) charismatice) participativeAnswer b) supportive

When Ingmar appeals to your loyalty and friendship when asking for your support, he isa) using consultation techniquesb) using ingratiation techniquesc) making an inspirational appeald) making a personal appeale) using legitimating tacticsAnswer d) make a personal appeal

Sheila is a very demanding boss to the extent that she regularly requires employees to work overtime without pay "or else". This is an example ofa) intimidationb) harassmentc) reward powerd) political behavioure) workplace bullyingAnswer e) workplace bullying

When Federico, from accounting, presents a large amount of data, using it to make a logical presentation of ideas, he is utilizinga) inspirational appealsb) rational persuasionc) legitimating tacticsd) consolation tacticse) personal appealAnswer b) rational persuasion

Activities that influence the distribution of advantages and disadvantages within the organization area) political behaviourb) tactical planning activitiesc) power brokering activitiesd) strategic planning activitiese) human resourcesAnswer a) political behaviour

Political behaviour isa) outside one's specified job requirementsb) not related to powerc) seen only in large organizationsd) counterproductive to individual goalse) part of each job requirementAnswer a) outside one's specified job requirements

Your colleagues at work are constantly talking about the VP in your area. He is feared throughout the organization as a ruthless man who is not to be antagonized. It is necessary for you to bring him a report and you are very nervous about having to deal with him.The VP's major base of power seems to bea) coerciveb) reward

Page 22: api.ning.comapi.ning.com/.../LatestNew2.docx  · Web viewNorms develop in one or more of four ways. Which of the following is not one of these ways? a) explicit statements made by

c) positiond) referente) expertAnswer a) coercive

When Sophia, your co-woker, gets others involved to support her in attaining her objectives, she isa) using coalition tacticsb) making an inspirational appealc) using ingratiation tacticsd) making a personal appeale) using consultation taticsAnswer a) using coalition tactics

When Woody, your co-worker, demands and threatens you to support his objectives, he isa) using coalition tacticsb) making an inspirational appealc) making a personal appeald) using pressure tacticse) using exchange tacticsAnswer d) using pressure tactics

If a person decides to employ a coercive base of power, he or she would be apt toa) make things difficult for peopleb) have the right to expect you to comply with legitimate requestsc) have the experience and knowledge to earn your respectd) have the data or knowledge that you neede) give special benefits or rewards to peopleAnswer a) make things difficult for people

Norma is notorious for doing favours for her co-workers so that they will "owe" her favours later on when she really needs them. Norma isa) managing impressionsb) building coalitionsc) influencing peopled) using informatione) creating obligationsAnswer e) creating obligations

How do individuals translate their bases of power into specific, desired action?a) implementation tacticsb) pressure tacticsc) empowerment tacticsd) influence tacticse) winning friendsAnswer d) influence tactics

Lisa works in the HR Department, and everyone tries todo favours for her because she has the ability to give special benefits to employees like free corporate tickets to hockey games. Lisa possessesa) legitimate powerb) information powerc) reward powerd) referent powere) coercive powerAnswer c) reward power

Yanni has been with the organization for 25 years, so even though he hasn't ever been promoted, he is held in high regard, and co-workers often defer to his judgement because of his experience and vast knowledge. Yanni possesses

Page 23: api.ning.comapi.ning.com/.../LatestNew2.docx  · Web viewNorms develop in one or more of four ways. Which of the following is not one of these ways? a) explicit statements made by

a) no powerb) referent powerc) expert powerd) legitimate powere) information powerAnswer d) expert power

Sexual harassment is frequently concerned with power abuseAnswer T

The degree of employee empowerment can be better understood by examine job content and job contextAnswer T

Even though he isn't very high up on the company's hierarchy, Cam is powerful since has been around for a long time and he has knowledge that no one else has in the organization .This is known as referent powerAnswer F

Regardless of the direction of influence, the most popular power tactic is the use of rational persuasionAnswer T

Donny is in the habit of offering his co-workers favours if they support him during staff meetings. This is known as an exchange tactic.Answer T

Resource in organizations are limited, which often turns potential conflict into real conflictAnswer T

Most businesses tend to develop multiple suppliers instead of just using one to avoid giving one supplier too much powerAnswer T

Which of the following principles should not be included in an individual's time management strategy to manage stress?a) accommodating appointments and/or tasks in order they are receivedb) making daily lists of activities to be accomplishedc) prioritizing activities by importance and urgencyd) knowing your personal energy cycle so that you handle the most demanding parts of your job during the high part of your cyclee) scheduling activities according to the priorities setAnswer a) accommodating appointments and/or tasks in order they are received

In contrasting values and attitudes, which of the following is true?a) Values are more stable than attitudesb) there is no significant correlation between the twoc) Values are based on diversity issuesd) they come from different sourcese) attitudes are more stable than valuesAnswer a) Values are more stable than attitudes

According to research conducted by Dr. Selye, stress occursa) in 3 stages: alarm, resistance, and exhaustingb) more frequently on a given day than people havec) as a result of lower educational levels among employeesd) in 4 stages: notification, alarm, resistance, and exhaustione) only when people are not feeling confident in themselvesAnswer e) only when people are not feeling confident in themselves

An Angus Reid poll which surveyed Canadians to find out their attitudes towards their jobs and workplaces determined that almost 40% or the respondents stated that one of the biggest barriers to job satisfaction was

Page 24: api.ning.comapi.ning.com/.../LatestNew2.docx  · Web viewNorms develop in one or more of four ways. Which of the following is not one of these ways? a) explicit statements made by

a) the nature of work itselfb) bureaucracy and red tapec) management practicesd) poor working conditionse) lack of recognitionAnswer b) bureaucracy and red tape

Tyrone always liked to "play by the rules" in life, and he place a lot of emphasis on discipline and order. He is likely a member of which of the following demographic groupsa) baby boomersb) echo boomersc) generation Xd) elderse) generation YAnswer d) elders

Canadians have certain values that Americans wish for such as diversity and fairness over divisiveness and racism, as well as the quality and consistency of our education. All of this leads Jack to conclude thata) Canadians values appear to be somewhat inferior to America valuesb) Americans appear to be more suited to working in teamsc) Canadians alue individuality and individual performance more than Americansd) Americans seem to follow the directives of the managers moree) Canadian and American workplaces will likely look and operate somewhat differentlyAnswer e) Canadian and American workplaces will likely look and operate somewhat differently

Li has recently arrived in Spain from Indonesia, and was very surprised to discover how little emphasis was placed on encouraging and rewarding individuals for being fair, altruistic, generous, and kind to others. Thus, in her view, Spain lackeda) assertivenessb) a humane orientationc) a future orientationd) uncertainty avoidancee) power distanceAnswer b) a humane orientation

Jack found that some corporations choose to highlight the value of diversity becausea) they feel it is an important strategic issue and goalb) Financial planning becomes easier and more logicalc) traditional business practices can best be served by diverse talentsd) Planning is likely to be less complex since everyone accepts diversity valuese) Organizational structures can be re-engineered more logicallyAnswer a) they feel it is an important strategic issue and goal

An employee who actively helps colleagues, takes minimal breaks, and alerts other to work-related problems isa) an organizational citizenb) high in job satisfactionc) high in job stabilityd) low in organizational loyaltye) high in organizational commitmentAnswer a) an organizational citizen

Norma has been working for the same organization for 25 years. She has worked there so long that she now feels an obligation to stay with this organization for the rest of her career. Norma is exhibitinga) normative commitmentb) continuance commitmentc) affective commitmentd) citizenship commitmente) personal commitment

Page 25: api.ning.comapi.ning.com/.../LatestNew2.docx  · Web viewNorms develop in one or more of four ways. Which of the following is not one of these ways? a) explicit statements made by

Answer a) normative commitment

Changing attitudes and organizational commitment are factors that will need to be defined and addressed. Jack has concluded that organizational commitment isa) a valid predictor for employee attitudes and valuesb) less universally understood by many individuals than job satisfactionc) probably a better predictor of employee turnover than job satisfactiond) not a factor which can be used to predict job satisfactione) the same as job involvement and can be used to predict successAnswer c) probably a better predictor of employee turnover than job satisfaction

According to GLOBE, which of the following countries scored low on uncertainty avoidance?a) Franceb) Hungaryc) Japand) Canadae) ChinaAnswer b) HungaryMGT502 Organizational Behaviour Solved MCQs  Set 2

What are personality traits?a) individual characteristics that are based solely on environmental factorsb) fleeting characteristics that describe an individual's behaviourc) enduring characteristics that describe an individual's behaviourd) individual characteristics that are based solely on hereditye) individual characteristics that are based solely on situational conditionsAnswer c) enduring characteristics that describe an individual's behaviour

When an individual displays different behaviours in different situations, this is termeda) flexibilityb) consensusc) continuityd) integritye) distinctivenessAnswer e) distinctiveness

The tendency for individuals to attribute their own success to internal factors while putting the blame for failures on external factors is calleda) contrast effectb) self-serving biasc) stereotypingd) projectione) the halo effectAnswer b) self-serving bias

Ava has a real ability to never give up, to persist in what she is trying to accomplish in the face of setbacks and failures. This is an example of hera) emotional intelligenceb) personalityc) moodsd) emotional dissonancee) self-esteemAnswer a) emotional intelligence

The unfounded belief that younger workers are more appealing than older workers is an example ofa) stereotypingb) contrast effects

Page 26: api.ning.comapi.ning.com/.../LatestNew2.docx  · Web viewNorms develop in one or more of four ways. Which of the following is not one of these ways? a) explicit statements made by

c) projectiond) prejudicee) the halo effectAnswer d) prejudice

When a fire truck's siren was heard, everyone ran to the window to see what was happening. That everyone responded in the same way is an example of which aspect of Attribution Theory?a) contrast effectb) consistencyc) selective perceptiond) distinctivenesse) consensusAnswer e) consensus

When a person stereotypes another person, the key thing she/he is doing is making generalizations based upona) the values of the other personb) a predefined group to which the other person belongsc) a key strength of the other persond) a key weakness of the other persone) the speaking style of the other personAnswer b) a predefined group to which the other person belongs

Sally is normally a very calm and composed manager, but the other day she sounded very visibly upset as evidenced by her raised voice on the telephone. Her staff heard it, and were trying to figure out what caused her to be upset. This is an example ofa) attribution theoryb) the Halo Effectc) stereotypingd) contrast effectse) projectionAnswer a) attribution theory

Which of the following is true about our perceptions of a target?a) targets are usually perceived in isolationb) individual perception varies little between different culturesc) Expectations do not distort individual perceptiond) Motion, sounds, size of a target have little effect in how individuals shape they way we perceive ite) timing of events and occurrences may imply a dependence, which may or may not be trueAnswer e) timing of event and occurrences may imply a dependence, which may or may not be true

There are many examples of shortcuts we make in judging others. Which shortcut below is least likely to be an error in perception about a new employee named Jo?a) knowing how good Jo will be because Jo has the same degree I haveb) using my ability to respond to individual difference between Jo and myselfc) using "high intelligence" of Jo to make a general impression about Jo's effectiveness on the jobd) deciding that Jo wants challenge and change since thats what I wante) letting my own attitudes and interests influence my impression of JoAnswer b) using my ability to respond to individual difference between Jo and myself

The evidence demonstrates that decision accuracy is the same for high- and low-risk-taking managersAnswer T

Negative information exposed early in the interview tends to be more heavily weighted than if the same information were conveyed laterAnswer T

People who see their lives as being controlled by outside forces have been called externals.Answer T

Page 27: api.ning.comapi.ning.com/.../LatestNew2.docx  · Web viewNorms develop in one or more of four ways. Which of the following is not one of these ways? a) explicit statements made by

Donna "wears her emotions on her sleeve" such that when she has trouble at work, it effects her performance. Thus is known as affective events theory.Answer T

Although intuitive thinkers (NT) make up only 5 percent of the population, a recent study of contemporary business people who created super-successful firms found that all of these individuals were NTs.Answer T

Proactive personalities can have both negative and positive impact depending on the organization and situational conditions.Answer T

Zack is always moving and appears to be impatient. He prefers work to leisure and seems obsessed with numbers. Zack is probably a Type A.Answer T

Tina is very cooperative, good-natured, and trusting. Thus, she scores high on conscientiousness according to the Big Five Model.Answer F

Felt emotions are the same as the emotions displayed in public.Answer F

Kerry tends to only remember what she wants to hear. This is known as selective perception.Answer F

Sam follows the practise in his department of periodically shifting his employees from one task to another in order to give them some variety and the opportunity to learn new skills. This practise is known asa) job designb) job enrichmentc) job enlargementd) job satisfactione) job rotationAnswer e) job rotationSome studies indicate that money is not the top predictor of employee satisfaction; insteada) recognition in the workplace mattered moreb) accomplishments in the workplace mattered morec) promotion in the workplace mattered mored) relationships in the workplace mattered moreAnswer d) relationships in the workplace mattered moreUnionized employees are typically paid on the basis ofa) performance and seniorityb) performance-based reviewc) seniority and job categoriesd) variable compensatione) seniority and productivityAnswer c) seniority and job categoriesOrganizations often "shoot themselves in the foot" when it comes to rewards by hoping for innovative thinking and risk taking but rewardinga) tighter controls over operations and resourcesb) the best team membersc) technical achievementsd) quarterly year earningse) proven methods and not making mistakesAnswer e) proven methods and not making mistakesNunavut has had great difficulty finding doctors willing to commit to serving the territory for more than short periods of time. The problem could be solved bya) job enlargement

Page 28: api.ning.comapi.ning.com/.../LatestNew2.docx  · Web viewNorms develop in one or more of four ways. Which of the following is not one of these ways? a) explicit statements made by

b) job enrichmentc) gainsharingd) job sharinge) job redesignAnswer d) job sharingIf jobs score high on motivation potential, the MPS model predicts that what will be positively affected?a) teamwork, collaboration and achievementb) motivation, performance and satisfactionc) empowerment, involvement, and developmentd) teamwork, performance, and satisfactione) responsibility, growth, and commitmentAnswer b) motivation, performance and satisfaction

Variable-pay or pay-for-performance programs operate reward programs as three levelsa) individual, group, and communityb) individual, departmental, and organizationc) individual, group, and organizationald) individual, group, and manageriale) individual, departmental, and communityAnswer c) individual, group, and organization

When using bonuses, managers should be mindful ofa) not recognizing friends within the companyb) potential unexpected behaviours arising when employees try to ensure they will receive bonusesc) the size of the bonus in relation to the event recognizedd) the amount, so that the bonus does not negatively effect the profits of the companye) very little, the bonus is just so effectiveAnswer b) potential unexpected behaviours arising when employees try to ensure they will receive bonuses

Job enrichment increases the degree to which workers control thea) planning, motivation, and incentives of their workb) ability to communicate with managers within the workplacec) hours and pay of their jobd) number of motivational incentives in the workplacee) planning, execution, and evaluation of their workAnswer e) planning, execution, and evaluation of their work

If a hard-working athlete or student with modest abilities consistently outperformed amore gifted, but lazy rival, this indicates thata) if either ability or motivation is inadequate, performance will be negatively affectedb) if both ability and motivation are inadequate, performance will be negatively affectedc) if either ability or motivation is inadequate, performance will be negatively affectede) if either ability or compensation is inadequate, performance will be negatively affectedAnswer a) if either ability or motivation is inadequate, performance will be negatively affected

Job enlargement isa) giving an employee a title which suggest he or she has more responsibility, but in fact does notb) expanding a job horizontallyc) expanding a job verticallyd) giving an employee more tasks while paying at the same levele) a result of downsizing, as employees must cover the duties of the departed employeeAnswer b) expanding a job horizontally

Profit sharing plansa) are company-established benefit plans in which employees acquires stock as part of their benefitsb) hive employees the right to buy stock in the company at a later date for a guaranteed pricec) focus on past financial resultsd) give employees the right to buy stock in the company at a later date for the market price at that time

Page 29: api.ning.comapi.ning.com/.../LatestNew2.docx  · Web viewNorms develop in one or more of four ways. Which of the following is not one of these ways? a) explicit statements made by

e) focus on future financial resultsAnswer c) focus on past financial results

The future of telecommuting is uncertain as there are still many unanswered questions regarding its advantages and disadvantagesAnswer T

Giving rewards for things that were previously done for intrinsic motivation will decrease motivationAnswer T

Job rotation increases skill varietyAnswer T

Motivation theories only give vague ideas of how money relates to individual motivationAnswer T

Studies have shown recognition to be the most powerful workplace motivatorAnswer T

Individuals respond differently to rewards depending upon what culture they come fromAnswer T

Employees in a gainsharing plan can recceive incntive awards even when the organization isn't profitableAnswer T

Performance evidence looking at the impact of flex-time does not stack up favourablyAnswer T

A body shop worker who sprays paint 8 hours a day is an example of a job with high skill variety characteristicsAnswer F

Organizational structure is made up of key elements. Which of the following is not one of these elements?a) formalizationb) coordinationc) decentralizationd) departmentalizatione) centralizationAnswer b) coordination

The degree to which tasks are subdivided into separate jobs is termeda) work specializationb) task structurec) departmentalizationd) formalizatione) span of controlAnswer a) work specialization

Work specialization is the same asa) span of controlb) job descriptionc) job groupingd) unity of commande) division of labourAnswer e) division of labour

Traditionally, managers have viewed which of the following as an unending source of increased productivity?

Page 30: api.ning.comapi.ning.com/.../LatestNew2.docx  · Web viewNorms develop in one or more of four ways. Which of the following is not one of these ways? a) explicit statements made by

a) delegationb) deparmentalizationc) work specializationd) centraliatine) formalizationAnswer c) work specialization

Which type of departmentalization achieves economies of scale by placing people with common skills and orientations into common units?a) geographicb) processc) technicald) functionale) productAnswer d) functional

The office supply firm, which has three departments to service retail, wholesale, and government customers, is practising which type of departmentalization?a) geographicb) productc) customerd) processe) functionalAnswer c) customer

The unbroken line of authority that extends from the top of the organization to the lowest echelon and clarifies who report to whom is termed:a) chain of commandb) authorityc) unity of commandd) power and authority structuree) powerAnswer a) chain of command

Which of the following refers to the number of subordinates a manger can efficiently and effectively direct?a) unity of commandb) chain of commandc) organizational chartd) structure of commande) span of controlAnswer e) span of control

Which one of the following components of organizational structure specifically defines where decisions are made?a) centralizationb) complexityc) formalizationd) technologye) departmentalizationAnswer a) centralization

What is a major difference between a mechanistic model and an organic model?a) one is characteristic of the service industryb) one involves new products and servicesc) they vary greatly in organizational sized) one has high formalizatione) one is for technological firmsAnswer d) one has high formalization

Page 31: api.ning.comapi.ning.com/.../LatestNew2.docx  · Web viewNorms develop in one or more of four ways. Which of the following is not one of these ways? a) explicit statements made by

Which of the following is consistent with a simple structure?a) high departmentalizationb) high horizontal differentiationc) high formalizationd) high centralizatione) high vertical differentiationAnswer d) high centralizationThe matrix structure combines thea) functional and simple structuresb) organic and mechanistic structuresc) simple and product structuresd) bureaucracy and mechanistic structurese) functional and product structuresAnswer e) functional and product structures

Which of the following problems is mot likely to occur in a matrix structure?a) a loss of economies of scaleb) decreased coordination effortsc) employees receiving conflicting directivesd) decreased employee motivatione) decreased response to environmental changeAnswer a) a loss of economies of scale

Which term refers to how an organization transfers its inputs to outputs?a) environmentb) innovationc) structured) technologye) strategyAnswer d) technology

Organization have different structures, but the structure has little bearing on employee attitudes and behaviourAnswer F

Work specialization can be carried too far and may result in employee boredom, stress, and absenteeismAnswer T

Organizations may choose only one of the forms of departmentalizationAnswer F

A decentralized organization is more likely to result in a feeling of alienation by employees than a centralized organizationAnswer F

Mechanistic structures are decentralizedAnswer F

The bureaucracy is characterized by highly routine tasks, formalized rules, and centralized authorityAnswer T

Effective teams need to work together and take collective responsibility to complete significant tasks.Answer T

Once a group's direction is set,it is re-examined frequently in the first half of the group's life and is likely to be altered.Answer F

Role expectation can be defined as a certain attitude or behaviour consistent with a role.

Page 32: api.ning.comapi.ning.com/.../LatestNew2.docx  · Web viewNorms develop in one or more of four ways. Which of the following is not one of these ways? a) explicit statements made by

Answer F

"Is this group ready to decide about this?" This question is an example of consensus testing.Answer T

A group must have at least two members who are interdependent with a common goal.Answer F

Conformity primarily leads to negative behaviour.Answer F

Norming is characterized by camaraderie.Answer T

Effective teams have a common and meaningful purpose that provides direction, momentum, and commitment for members.Answer T

For multi-team systems to succeed, different teams need to coordinate their efforts to produce a desired outcomeAnswer T

Groups that work closely together toward a common objective and are accountable to one another are called teamsAnswer T

Members in a group are not necessarily interdependentAnswer T

Joan was appalled by how much conflict there was among the members of her team. Although the team had only been in existence for a short period of time, she was troubled. But when she discussed this with her team leader, he indicated that this was typical of groups at their forming stage. Is the response of Joan's leader correct?Answer F

Role conflict is defined as a situation in which an individual has certain attitudes and behaviour that are disagreeableAnswer T

Most conflicts are resolved by the completion of the first stage in a group's developmentAnswer F

"I think what you started to say is important, Jack. Please continue." This request is an example of an encouraging function to maintain good relations.Answer T

"Let's set up an agenda for discussing each of the problems we have to consider." This suggestion is an example of a harmonizing function to maintain good relations.Answer F

Virtual teams are typically dependent on work being completed face to face.Answer F

The Punctuated-Equilibrium Model characterizes groups as exhibiting long periods of inertia interspersed with brief revolutionary changes triggered primarily by members' awareness of time and deadlines.Answer T

All groups are teams but not all teams are groupsAnswer F

Virtual teams can suffer from the absence of paraverbal and nonverbal cues as well as social isolation.Answer T

Page 33: api.ning.comapi.ning.com/.../LatestNew2.docx  · Web viewNorms develop in one or more of four ways. Which of the following is not one of these ways? a) explicit statements made by

The process that adapts employees to the organization's culture is called:a) socializationb) formalizationc) indoctrinationd) orientatione) confirmationAnswer a) socialization

You are new to an organization and do not really know what to expect about the socialization process. you are a recent MBA and have an undergraduate degree in computer science. Your new firm is a software development company with an emphasis on the health care industry.The stage in which you will probably become detached from previous assumptions and replace those with the organization considers desirable is called:a) social facilitation stageb) replacement stagec) metamorphosis staged) encounter stagee) prearrival stageAnswer d) encounter

Masterson College is a small liberal arts women's college in North Carolina. The founds of the college were Baptist and were committed to the idea that a liberal arts education was the best preparing for life-long learning. Within the last two decades, the business department has become of the larger departments on campus. The faculty of the business department are committed to liberal arts education but are also committed to finding employment for their graduates.The commitment to finding employment for graduates is part of thea) dominent cultureb) instructional planc) weak cultured) mission statemente) subculture of the business departmentAnswer e) subculture of the business department

Which of the following is not an example of what an organizational culture does?a) enhances social system stabilityb) conveys identity for organization membersc) establishes boundaries between organizationsd) outlines the formal hierarchye) acts as a behavioural control mechanismAnswer d) outlines the formal hierarchy

If there is a basic conflict between the individual's expectations and the organization's expectations, the employee is most like to be disillusioned and quit during which stage?a) anxietyb) encounterc) metamorphosisd) adjustmente) prearrivalAnswer b) encounter

Which of the following is not considered to be a characteristic of organizational culture?a) outcome orientationb) people orientationc) innovationd) instabilitye) aggressiveness

Page 34: api.ning.comapi.ning.com/.../LatestNew2.docx  · Web viewNorms develop in one or more of four ways. Which of the following is not one of these ways? a) explicit statements made by

Answer d) instability

Which of the following does not sustain an organizational culture?a) promotion proceduresb) selection practisesc) socializationd) formalizatione) top managementAnswer d) formalization

Smaller cultures within an organization, typically define by departmental designations and geographical separation, are often called:a) strong culturesb) mixed culturesc) dominant culturesd) departmental entitiese) subculturesAnswer e) subcultures

How does "top management'' have a major impact on an organization's culture?a) providing appropriate trainingb) maintaining traditions of past practisesc) ensuring a proper match of personal and organizational valuesd) establishing norms that filter down through the organizatione) socializing the applicantAnswer d) establishing norms that filter down through the organization

Culture has a boundry-defing role; it creates distinctions between one organization and othersAnswer T

Given the relative power asymmetries it is unlikely that a subculture could undermine the dominant cultureAnswer F

A strong culture can be a liabilityAnswer T

The size and layout of an organization's corporate headquarters exemplifies material symbolsAnswer T

Three forces play a particularly important part in sustaining culture: selection practises, promotion policies, and socialization methods.Answer F

A strong culture provides a supportive atmosphere for diversityAnswer F

Organizational culture can be transmitted to employees by stories, rituals, material symbols, and languageAnswer T

Culture sets the tone for how an organization operated and how individual members within the organization interactAnswer F

The degree to which organizational activities emphasize maintaining the status quo in contrast to growth is termed stabilityAnswer T

The degree to which management focuses on results rather than on techniques and processes termed outcome orientationAnswer T

Page 35: api.ning.comapi.ning.com/.../LatestNew2.docx  · Web viewNorms develop in one or more of four ways. Which of the following is not one of these ways? a) explicit statements made by

All organizations within an industry use the terminology designated by the industry so that their employees can move freely from organization to organizationAnswer F

Research studies in diverse settings confirm that functionality of conflict, particularity as it relates to productivity.Answer T

Dysfunctional conflicts hinder a group's performanceAnswer T

Arbitration is a process in which two or more parties exchange goods or services and attempt to agree upon the exchange rate for them.Answer F

Many open and violent conflicts in work groups are functionalAnswer F

Teams experiencing affective conflict have poorer decisions and lower levels of acceptance of the decisionAnswer T

An individual's target point represents the lowest outcome he or she would accept during a negotiation processAnswer F

Edward is thinking of reorganizing his department so that Fred and Sarah do not have to work together, as they don't seem to get along. Whenever they have a disagreement about a task, it always seems to degenerate into emotional attacks against each other. This is known as cognitive contact.Answer F

Integrative bargaining is the strategy most used in today's intraorganizational problem-solving behaviourAnswer F

Choosing avoidance as a conflict management strategy is appropriate when you find out that you are wrongAnswer F

Compromise is when each party to a conflict gives up something of valueAnswer T

Avoiding encompasses both assertive and uncooperative behavioursAnswer F

Conflict always improve the quality of group decision makingAnswer F

Attempting to convince another that your conclusion is right and his or her conclusion is mistaken is an example of the forcing conflict-handling strategy.Answer T

Across cultures, people have different ideas about the appropriateness and effects of conflict.Answer T

The only desired outcome of a conflict is an equitable and fair agreement between the parties involved.Answer F

Ron felt that the best approach to resolving the conflict between two of his staff members was to merge both of their perspectives, as they both seemed valid to him. This approach to conflict management is known as problem solvingAnswer T

Page 36: api.ning.comapi.ning.com/.../LatestNew2.docx  · Web viewNorms develop in one or more of four ways. Which of the following is not one of these ways? a) explicit statements made by

The smaller the group and the less specialized its activities, the great likelihood of conflictAnswer F

The major advantage of mediation over arbitration is that it always results in a settlementAnswer F

An accurate definition of a conflict is important as this will help to establish the type of outcomes, which might settle it.Answer F

Within conflict, accepting the will of the other party is termed yielding.Answer T

What is an organization's most common informal network?a) e-mailb) voice-mailc) bulletind) grapevinee) memoAnswer d) grapevine

Communication that takes place among members of the same work group isa) technical communicationb) vertical communicationc) occupational communicationd) lateral communicatione) downward communicationAnswer d) lateral communication

Ted was always very conscious of not invading his employees personal "space" when speaking to them by not getting too close when he was talking to hem. Ted was therefore paying attention toa) empathyb) kinesicsc) cultural barriersd) proxemicse) groupthinkAnswer d) proxemics

Which communication network does an unstructured committee best illustrate?a) horizontalb) wheelc) circled) interpersonale) all-channelAnswer e) all-channel

In dealing with cross-cultural communication, a manager might practise putting herself in the place of the employee, a method termeda) symbolic interactionb) empathyc) sympathyd) hypothesis testinge) semanticsAnswer b) empathyWhich of the following is not considered an organization?a) university

Page 37: api.ning.comapi.ning.com/.../LatestNew2.docx  · Web viewNorms develop in one or more of four ways. Which of the following is not one of these ways? a) explicit statements made by

b) military unitc) all 45 year olds in a communityd) municipal, provincial and federal government agenciese) churchAnswer c) all 45 year olds in a community

A(n) _________ is consciously coordinated social unit, composed of two or more people, that functions on a relatively continuous basis to achieve a common goal or set of goals.a) informal groupb) teamc) ethnic groupd) unite) organizationAnswer e) organizationhttp://www.vuzs.net

Looking in-depth at single situations in order to gain insights into organization behaviour relates to the examination ofa) survey studiesb) field studiesc) laboratory studiesd) case studiese) meta-analysisAnswer d) case studies

Giselle has introduced a new performance measure that focuses on an employee's efficiency and effectiveness. This is known as the employee'sa) productivityb) diversityc) ethicsd) culturee) performanceAnswer a) productivity

The behavioural science which underlies organizational cultures is know an a) psychologyb) anthropologyc) social psychologyd) sociologye) political scienceAnswer b) anthropology

Employee productivity, developing effective employees, and putting people first are challenges at the ______ levela) individualb) motivationalc) organizationald) groupe) ethicalAnswer c) organizational

The behavioural science which underlies work design and performance appraisal is known asa) sociologyb) social psychologyc) political scienced) psychologye) anthropologyAnswer d) psychology

Page 38: api.ning.comapi.ning.com/.../LatestNew2.docx  · Web viewNorms develop in one or more of four ways. Which of the following is not one of these ways? a) explicit statements made by

The approach to organization behaviour which suggests that its answers "depends upon the situation" is called thea) contingency approachb) behavioural approachc) field study approachd) mechanistic approache) organic approachAnswer a) contingency approach

Working with others and workplace diversity are challenges at the ____ levela) organizationalb) motivationalc) individuald) ethicale) groupAnswer e) group

The most significant contributions to OB with respect to implementing change and reducing barriers to its acceptance has been made bya) sociologyb) anthropologyc) political scienced) social psychologye) psychologyAnswer d) social psychology

Which of the following is an example of being an efficient organization or employee?a) being a sales person who acquires the most clients of anyone in the companyb) operating a hospital at the lowest possible cost while achieving higher outputc) being an instructor who teaches larger classes than his fellow colleaguesd) being the most pleasant real estate broker in Torontoe) being a telemarketer who makes the required number of calls at the end of the dayAnswer b) operatin ga hospital at the lowest possible cost while achieving higher output

If today's managers positively address and manage workforce diversity, which of the following increase?a) miscommunicationsb) creativity and innovationc) employee turnoverd) ineffective decision makinge) interpersonal conflictsAnswer b) creativity and innovation

A major theme underlying the process of organizational behaviour is the belief that human behaviour isa) opaqueb) knowablec) unpredictabled) intuitivee) instinctualAnswer b) knowable

Workplace diversity occurs because organizations are becoming more homogeneous Answer F

The term organization generally refers to business firms but excludes government, social service agencies and other public organizationsAnswer F

If diversity is ineffectively managed, high turnover and miscommunication can result, creating an environment, which hinders

Page 39: api.ning.comapi.ning.com/.../LatestNew2.docx  · Web viewNorms develop in one or more of four ways. Which of the following is not one of these ways? a) explicit statements made by

achievement of organizational goalsAnswer T

The behaviour of individuals in a structured situation is similar to their behaviour when they are in unstructured situationsAnswer F

Ethics can inform us whether our individual actions are right or wrong; however, organization and group actions are too complex for ethics to applyAnswer F

Organizational behaviour is best suited for managers in large corporations rather than small businesses and entrepreneurial enterprisesAnswer F

The challenge for organizations is to accommodate diverse groups of people by addressing their different lifestyles, family needs, and work styleAnswer T

SMART stands fora) specific, unfixed, attainable, results-oriented, and time-boundb) specific, measurable, inaccessible, results-oriented, and time-boundc) specific, measurable, attainable, results0oriented, and time-boundd) specific, measurable, attainable, results-oriented, and time-bounde) equivocal, measurable, attainable, results-oriented, and time-boundAnswer d) specific, measurable, attainable, results-oriented, and time-bound

You graduated from college two years ago and began working at Hampstead Electronics. You have received good performance evaluations and a raise. You just found out that a recent college graduate with no experience has been hired at a higher salary than you are now making.Which theory will you probably use in evaluating how you regard the situation?a) equityb) performance-rewardc) expectancyd) reinforcemente) goal settingAnswer a) equity

Your department is made up of people who are very different in their lifestyles and their stages of life. Mary is a 23-year-old single parent who is working for minimum wage. Jonathan is 60 years old, extremely wealthy and works because he enjoys it. Jane is single, 45 years old and has few interests outside the office. You have decided to attempt to apply Maslow's hierarchy of needs to motivate each of these individuals.You would expect that Mary is trying to satisfy which need?a) esteemb) safetyc) self-actualizationd) sociale) physiologicalAnswer e) physiological

Your organization has decided to institute a management by objectives program. The people in your department have asked you to explain the program and help them understand how it will affect them.Which of the following is an example of an MBO objective?a) quality of all products should increase substantiallyb) telephone orders should be processed promptlyc) company costs should be decreased as much as possibled) sales should be increased by two percente) new customers should be brought in on a regular basis

Page 40: api.ning.comapi.ning.com/.../LatestNew2.docx  · Web viewNorms develop in one or more of four ways. Which of the following is not one of these ways? a) explicit statements made by

Answer d) sals should be increased by two percent

Performance evaluations done by one's associates is calleda) immediate subordinate(s) reviewb) immediate superior's reviewc) the comprehensive approach: 360 degree evaluationd) combined subordinate/superior reviewe) peer reviewAnswer e) peer review

Jason faced a real uproar recently from his employees after they received their annual pay increases. The main complaint was that his employees didn't think that the process by which the increases were awarded was fair. This is an example ofa) interactional justiceb) operant justicec) procedural justiced) distributive justicee) equitable justiceAnswer c) procedural justice

The premise that higher level needs can emerge before lower level needs are met is part of a) ERG theoryb) process theoryc) Maslow's theoryd) expectancy theorye) valence theoryAnswer a) ERG theory

According to cognitive evaluation theory, pay should not be directly related to job performance.Answer T

Julie wishes to apply Maslow's hierarchy in her workplace. She is interested in how various employee needs are met by the organization.The organization will find that employees' self-actualization needs are the easiest to satisfyAnswer F

Self-efficacy refers to an individual's belief that he or she is capable of performing.Answer TAccording to Alderfer, once an employee's existence needs are substantially satisfied, the employee moves on to his or her relatedness needs.Answer T

Within McClelland's theory of needs, the need for power is the drive to excel, and to achieve in relation to a set of standards.Answer F

The most important thing to John after 15 years with the same company is his job title and public recognition. This indicates that he is at the self-actualization stage of Maslow's hierarchy.Answer T

Institutions such as prisons and the military are unlikely to embrace Theory XAnswer F

Xavier is considered to be a very sensitive manager when it come to the issues facing his employees on a personal level. This means that he scores high with them in terms of distributive justice.Answer F

Operant conditioning assumes behaviour is influenced by reinforcement or lack of reinforcement brought about by the consequences of behaviour

Page 41: api.ning.comapi.ning.com/.../LatestNew2.docx  · Web viewNorms develop in one or more of four ways. Which of the following is not one of these ways? a) explicit statements made by

Answer T

Hygienic factors usually lead to job satisfaction when present in a jobAnswer F

The four ingredients common to MBO programs are goal specificity, participative decision making, an implicit time period, and performance feedbackAnswer F

Michealson suggests that researchers have a moral obligation to make workplaces better for employeesAnswer T

Tom is of the belief that the more effort that he puts into his job, the better he will perform. This is called expectancyAnswer T

MGT 502 Organizational Behaviour 100 S